subject
Physics, 03.11.2020 17:20 nessabear9472

A flat loop of wire consisting of a single turn of cross-sectional area 7.10 cm2 is perpendicular to a magnetic field that increases uniformly in magnitude from 0.500 T to 1.50 T in 1.03 s. What is the resulting induced current if the loop has a resistance of 1.60

ansver
Answers: 2

Another question on Physics

question
Physics, 21.06.2019 20:20
How much work would a child do while puling a 12-kg wagon a distance of 3m with a 22 n force directed 30 degrees with respect to the horizontal? (a) 82j (b) 52j (c) 109j (d) 95j
Answers: 2
question
Physics, 22.06.2019 15:00
How to calculate voltage drop in a series circuit
Answers: 1
question
Physics, 22.06.2019 22:30
I’ve been stuck on this for a while
Answers: 1
question
Physics, 22.06.2019 23:40
A2.50 × 105 w motor is used for 26.4 s to pull a boat straight toward shore. how far does the boat move toward shore if a force of 4.20 × 104 n is applied by the motor?
Answers: 2
You know the right answer?
A flat loop of wire consisting of a single turn of cross-sectional area 7.10 cm2 is perpendicular to...
Questions
question
Social Studies, 26.07.2019 14:10
Questions on the website: 13722367